Good bye 2017 2017

Find last 3 digits in decimal representation of:

7 17 017 2017 \Large 7^{{17}^{{017}^{2017}}}


The answer is 207.

This section requires Javascript.
You are seeing this because something didn't load right. We suggest you, (a) try refreshing the page, (b) enabling javascript if it is disabled on your browser and, finally, (c) loading the non-javascript version of this page . We're sorry about the hassle.

1 solution

Chew-Seong Cheong
Dec 14, 2017

Let N = 7 1 7 1 7 2017 N = 7^{17^{17^{2017}}} . We need to find N m o d 1000 N \bmod 1000 . Since 7, 17 and 2017 are primes, we can use Euler's theorem and the respective Carmichael lambda values are λ ( 1000 ) = 100 \lambda (1000) = 100 , λ ( 100 ) = 20 \lambda (100) = 20 and λ ( 20 ) = 4 \lambda (20) = 4 . Then, we have:

N 7 1 7 1 7 2017 m o d 4 m o d 20 m o d 100 (mod 1000) 7 1 7 1 7 1 m o d 20 m o d 100 (mod 1000) 7 1 7 17 m o d 100 (mod 1000) 7 1 7 17 m o d 100 (mod 1000) 7 ( 20 3 ) 15 ( 1 7 2 ) m o d 100 (mod 1000) 7 3 15 ( 89 ) m o d 100 (mod 1000) 7 3 ( 10 1 ) 7 ( 11 ) m o d 100 (mod 1000) 7 3 ( 69 ) ( 11 ) m o d 100 (mod 1000) 7 77 (mod 1000) By modular inverse (see note), 207 (mod 1000) \large \begin{aligned} N & \equiv 7^{17^{17^{2017 \bmod 4} \bmod 20} \bmod 100} \text{ (mod 1000)} \\ & \equiv 7^{17^{17^1 \bmod 20} \bmod 100} \text{ (mod 1000)} \\ & \equiv 7^{17^{17} \bmod 100} \text{ (mod 1000)} \\ & \equiv 7^{17^{17} \bmod 100} \text{ (mod 1000)} \\ & \equiv 7^{(20-3)^{15}(17^2) \bmod 100} \text{ (mod 1000)} \\ & \equiv 7^{-3^{15}(89) \bmod 100} \text{ (mod 1000)} \\ & \equiv 7^{3(10-1)^7(11) \bmod 100} \text{ (mod 1000)} \\ & \equiv 7^{3(69)(11) \bmod 100} \text{ (mod 1000)} \\ & \equiv 7^{77} \text{ (mod 1000)} & \small \color{#3D99F6} \text{By modular inverse (see note),} \\ & \equiv \boxed{207} \text{ (mod 1000)} \end{aligned}


Note:

7 80 ( 50 1 ) 40 1 (mod 1000) 7 3 7 77 1 (mod 1000) 343 ( 7 77 ) 1 (mod 1000) 343 ( 7 ) 401 (mod 1000) 343 ( 207 ) 1 (mod 1000) 7 77 207 (mod 1000) \begin{aligned} 7^{80} & \equiv (50-1)^{40} \equiv 1 \text{ (mod 1000)} \\ \implies 7^3 7^{77} & \equiv 1 \text{ (mod 1000)} \\ 343(7^{77}) & \equiv 1 \text{ (mod 1000)} \\ 343(7) & \equiv 401 \text{ (mod 1000)} \\ 343(207) & \equiv 1 \text{ (mod 1000)} \\ \implies 7^{77} & \equiv 207 \text{ (mod 1000)} \end{aligned}

thank you for an excellent solution

Mrigank Shekhar Pathak - 3 years, 5 months ago

0 pending reports

×

Problem Loading...

Note Loading...

Set Loading...